LSAT and Law School Admissions Forum

Get expert LSAT preparation and law school admissions advice from PowerScore Test Preparation.

User avatar
 Dave Killoran
PowerScore Staff
  • PowerScore Staff
  • Posts: 5852
  • Joined: Mar 25, 2011
|
#59767
Complete Question Explanation
(The complete setup for this game can be found here: lsat/viewtopic.php?t=1753)

The correct answer choice is (A)

If K is within exactly two areas, K cannot be in area U, since area U is also within areas R and T. Because K also cannot be in area S from the game setup, K must be in areas R and T:

pt15_j95_g2_q8.png
Because K is in area T, from the third rule J cannot be in area T, and answer choice (A) is correct.
You do not have the required permissions to view the files attached to this post.

Get the most out of your LSAT Prep Plus subscription.

Analyze and track your performance with our Testing and Analytics Package.